staff and student guide to synchronous learning in year 4 2013

44
1 STAFF and STUDENT GUIDE to Synchronous Learning in Year 4 2013 Introduction...................................................................................................................................................2 Purpose ........................................................................................................................................................2 Other relevant activities and resources .........................................................................................................2 Brief description of Year 4 Synchronous Learning programme .....................................................................2 Timing ..........................................................................................................................................................3 Organisation .................................................................................................................................................3 Contact staff at sites .....................................................................................................................................3 Pedagogy .....................................................................................................................................................4 Outline of Synchronous Learning in 2013 .....................................................................................................5 Tutorial 1 Refresher of Clinical Methods .......................................................................................................7 Tutorial 2 Respiratory ...................................................................................................................................8 Tutorial 3 Cardiology ..................................................................................................................................11 Tutorial 4 Renal Medicine ...........................................................................................................................13 Tutorial 5 Gastroenterology ........................................................................................................................17 Tutorial 6 Neurology ...................................................................................................................................21 Tutorial 7 Endocrinology .............................................................................................................................23 Tutorial 8 Clinical Pathology .......................................................................................................................25 Tutorial 9 Rheumatology.............................................................................................................................29 Tutorial 10 Infectious Diseases (Medicine) ................................................................................................. 34 Tutorial 11 Haematological Cancer Presentations in the Community .......................................................... 36 Tutorial 12 Dermatology .............................................................................................................................40 Tutorial 13 Infectious Diseases (Surgery) ...................................................................................................41 Tutorial 14 ORL ..........................................................................................................................................42 Tutorial 15 Plastics .....................................................................................................................................44 Development of this Guide has been led by Prof Phillippa Poole in consultation with academic staff from the Formal Learning Subcommittee and in the Departments of Medicine, Surgery, Clinical Pharmacology, and Molecular Medicine and Pathology. Dr Susannah O’Sullivan has contributed significantly, especially to the development of the Rational Management programme. If you have feedback or queries, please discuss with the relevant academics or else directly with Prof Poole, email [email protected]. Details updated as at 22 July 2013

Upload: others

Post on 11-Feb-2022

2 views

Category:

Documents


0 download

TRANSCRIPT

1

STAFF and STUDENT GUIDE to Synchronous Learning in Year 4 2013

Introduction ................................................................................................................................................... 2 Purpose ........................................................................................................................................................ 2 Other relevant activities and resources ......................................................................................................... 2 Brief description of Year 4 Synchronous Learning programme ..................................................................... 2 Timing .......................................................................................................................................................... 3 Organisation ................................................................................................................................................. 3 Contact staff at sites ..................................................................................................................................... 3 Pedagogy ..................................................................................................................................................... 4 Outline of Synchronous Learning in 2013 ..................................................................................................... 5 Tutorial 1 Refresher of Clinical Methods ....................................................................................................... 7 Tutorial 2 Respiratory ................................................................................................................................... 8 Tutorial 3 Cardiology .................................................................................................................................. 11 Tutorial 4 Renal Medicine ........................................................................................................................... 13 Tutorial 5 Gastroenterology ........................................................................................................................ 17 Tutorial 6 Neurology ................................................................................................................................... 21 Tutorial 7 Endocrinology ............................................................................................................................. 23 Tutorial 8 Clinical Pathology ....................................................................................................................... 25 Tutorial 9 Rheumatology ............................................................................................................................. 29 Tutorial 10 Infectious Diseases (Medicine) ................................................................................................. 34 Tutorial 11 Haematological Cancer Presentations in the Community .......................................................... 36 Tutorial 12 Dermatology ............................................................................................................................. 40 Tutorial 13 Infectious Diseases (Surgery) ................................................................................................... 41 Tutorial 14 ORL .......................................................................................................................................... 42 Tutorial 15 Plastics ..................................................................................................................................... 44

Development of this Guide has been led by Prof Phillippa Poole in consultation with academic staff from the Formal Learning Subcommittee and in the Departments of Medicine, Surgery, Clinical Pharmacology, and Molecular Medicine and Pathology. Dr Susannah O’Sullivan has contributed significantly, especially to the development of the Rational Management programme. If you have feedback or queries, please discuss with the relevant academics or else directly with Prof Poole, email [email protected].

Details updated as at 22 July 2013

2

Introduction The reinvigoration of the MBChB curriculum from 2013, and the cohorting of students at four clinical campuses, each create opportunities for greater integration between formal learning activities and student learning in clinical environments. The three Year 4 Medicine clinical attachments: General Medicine (6 weeks), Specialty Medicine (6 weeks), and Geriatrics (4 weeks), which run sequentially in one half of the year for half the students at a clinical campus (and repeat in the other half year for the balance of the students). For practical purposes this is labelled as the ‘Medicine’ half year. The other half of the year is made up of attachments in Musculoskeletal (4 weeks), Emergency Medicine (3 weeks), General Surgery (6 weeks), Anaesthesiology (2 weeks), and Procedural Skills, and is labelled the ‘Surgery’ half year. Formal Learning in Year 4 has strong horizontal links to clinical experiences in Year 4, and vertical links with learning from earlier years, and to later medical practice. Further, Formal Learning has strong links to the newly-developed suite of 189 Clinical Scenarios that define the core knowledge and skills to be acquired in the programme. These may be found at http://mbchb.auckland.ac.nz/scenarios

There are three elements of Formal Learning:

1. Formal Learning Weeks on campus at Grafton 2. Asynchronous Formal Learning - modules for student to complete in their own time, and 3. Synchronous Formal Learning delivered at clinical campuses/ sites, involving a session for group (s)

of students at a particular campus at the one time.

Purpose This Guide is written to facilitate the effective delivery of Synchronous Learning (SL) across all sites.

Other relevant activities and resources

MBChB Guidebooks, handbooks and other resources: 2013 MBChB Year 4 Guidebook

Year 4 Clinical attachment handbooks

Year 3 Clinical Methods handbook

SOM Clinical Teacher handbook being developed by Prof John Kolbe

The Clinical Scenarios (~ 189) that form a skeleton for learning in the MBChB programme http://mbchb.auckland.ac.nz/scenarios Phase 1 materials e.g. lecture notes and the North-Nanson Red Book Brief description of Year 4 Synchronous Learning programme The SL programme is comprised of 15 two-hour sessions. Twelve of these are broadly ‘medical’ and three broadly ‘surgical’. Each session is delivered twice - to half of the students in the first six months, and to the other half in the second six months. This is for two reasons: to keep group sizes manageable; and to maximise the link between synchronous learning and clinical attachments.

Broadly, the format for the tutorials needs to be followed. Each tutorial has been developed by named lead staff in disciplines within the Departments of Medicine, Surgery, Clinical Pharmacology, and Molecular Medicine and Pathology, in conjunction with staff on the Formal Learning Subcommittee of the Board of Studies. The format takes into account prior learning, pedagogy, curriculum goals, and feasibility. This programme replaces the previous Core Topic teaching in the General Medicine attachment, some tutorials in Specialty Medicine, plus some Therapeutics and Lab Medicine teaching previously done in Campus Learning Weeks. It does not preclude other tutorials being offered at sites, provided these Synchronous Learning sessions are delivered as a priority.

3

Timing It has been agreed that default day for SL is a Tuesday afternoon. There may be a short break between sessions. On Tuesdays when there is no SL scheduled, this time is designated for students to use for other learning activities such as Asynchronous Learning and assignments, as part of the weekly student study half day.

At Waikato SL will be held on a Wednesday morning. At Waitemata, SL sessions will start at 1.30pm; at Auckland from 1.00pm- 3.30pm.

Organisation Student numbers for 2013 are:

Year 4 Site in 2013 Each half year Total Auckland 37 74 Middlemore 30 60 Waitemata DHB 25 50 Waikato/Rotorua 10 20 Total 102 204

There is an academic coordinator at each site in charge of SL, who may come from the Department of Medicine, given the number of ‘Medicine’ tutorials, and who is supported by a professional administrator. Between them, the coordinator and administrator:

• Confirm the programme with tutors. Tutors will mainly be on-site, but on occasions, a tutor may deliver at more than one site. Tutors at the sites need not necessarily be subspecialists, and should act as facilitators;

• Confirm the days, times, topics and location of SL and promulgate a detailed timetable to staff and students. Note: if resources allow, smaller groups may be employed.

• Confirm local timing of tutorial in week of 28th October; which can’t be on Tuesday (Progress Test) • Ensure rooms are booked (2.5 hours minimum); • Allocate two students to lead each SL session; • Provide the model answers to tutors if requested, NOT students; • Check sessions are delivered, and deal with feedback from staff and students, channelling it where

appropriate; • Work closely with coordinators of clinical attachments to make sure students are able to attend SL; • Liaise with the lead academic for the topic, or HOD, if there are issues with content or difficulty

getting staff for delivery.

Contact staff at sites

Waitemata Academic: Prof Martin Connolly / Dr Vinod Singh (Medicine), Mr Jonathan Koea (Surgery) Professional: Mere Vercoe

South Auckland Academic: Dr Roger Reynolds (Medicine), Mr Andrew MacCormick (Surgery) Professional: Maria Vitas

Auckland Academic: Prof Phillippa Poole (Medicine), Prof Bryan Parry (Surgery) Professional: Lindsay Richdale (Medicine), Christine Ganly (Surgery)

Waikato / Rotorua Academic: AProf John Conaglen (Medicine) Professional: Raewyn Wooderson

4

Pedagogy Having one designated study half day per week, every week, should make it easier for students to prioritise clinical, formal and individual learning activities during Year 4. Students are expected to attend SL sessions, to have read the materials and the scenarios prior to the tutorials, and be prepared to engage fully in the tutorial. SL sessions are, however, not compulsory.

The main aim of the SL sessions in Year 4 is to enable students to learn to apply their knowledge in clinical reasoning or problem solving. Clinical reasoning has been defined by K. Eva (2007) and others as “the ability to sort through a cluster of features presented by a patient and assign accurate diagnostic label/s so as to develop an appropriate management plan, including treatment.” Specifically, the focus of these sessions is largely on the diagnosis and management of common and important clinical conditions students will meet and manage as doctors. In general, they cover topics in only two of the Auckland MBChB curriculum domains, viz. Applied Science for Medicine, and Clinical and Communication Skills. Thus, they are largely clinical science tutorials. Please note that the Clinical Scenarios outline the broader context of this learning through inclusion of another three domains viz. Personal and Professional Skills, Hauora Māori and Population Health.

These are not to be didactic sessions. To aid application and deep learning, there is a student-led component to most tutorials. The nominated students should do any preparation necessary for the tutorials (e.g. check session details, find and present cases if needed, organise allocation of problems to fellow students, be prepared to propose or answer questions, review the relevant MBChB Clinical Scenarios, and liaise with tutors about aspects they want to cover in the session).

The tutor is largely to act as a facilitator and resource person, and is not expected to prepare a formal talk, except where specified. The tutor should familiarise themselves with the topics in the session, the model answers (available from lead academic or professional staff at sites), and ideally with the Clinical Scenarios; being prepared to point these out to the students. These scenarios detail the knowledge and skills expected by graduation in all domains.

Format The formats of the tutorials vary slightly. Many for organ systems follow the format below:

First Hour – Clinical Reasoning The two nominated students at the site will be pre-allocated the tasks of:

1. Each finding and presenting a case with (that specialty) problem, from their ward experiences; 2. Compiling questions from students relating to the cases or that specialty.

Students will need to communicate with each other in order to prepare for the tutorial. After the presentation of the two patient vignettes (max 15 minutes), the remainder of the hour will be spent on:

1. discussing relevant aspects of the two cases and covering student queries; 2. clinical reasoning in that specialty:

a. how typical or atypical were the presentations; b. specific history and exam findings, and techniques; c. common differential diagnoses; d. red flags, pitfalls; e. the role and interpretation of common investigations; f. choosing treatments/ delivery mechanisms.

3. addressing the broader curriculum domains of Hauora Māori and Population Health 4. covering other specific case-based topics raised by students.

Second Hour – Rational Management Students will work through paper cases and answer questions that illustrate important diagnostic and therapeutic principles in medical specialties. For other sessions such as Rheumatology and Clinical Pathology, the format for both hours is based on paper cases. It is suggested there is a short break between the two sessions.

5

Outline of Synchronous Learning in 2013 The generic programme is outlined below. The Tuesday dates have been added as the default time for SL is Tuesday afternoon. An early session of the year (Tut 1) is a refresher of the Clinical Method for all students, and may be combined for all students in Week 2 at smaller sites. After that, sites may vary the order of the other sessions depending on staff and room availability, and, if necessary alter day of the week.

Please NOTE:

• Waikato sessions will be run on a Wednesday. • At Auckland, tutorials start at 1300; at Waitemata they start at 1330. • *There is a clash with the Progress Test on October 29th for Tuts 11 and 13 in the second half of the

year. Sites need to confirm own arrangements, with options being a time on Wednesday, Thursday or Friday that week.

For students starting on the ‘Medicine’ half year

SL Medicine Topic Format Tut 1 Tues 19 Feb

Clinical Methods revision

Tut 2 Tues 26 Feb

Respiratory

Clinical Reasoning Rational Management

2 students allocated to present cases

Tut 3 Tues 5 Mar

Cardiology

Clinical Reasoning Rational Management

2 students allocated to present cases

Tut 4 Tues 12 Mar

Renal

Clinical Reasoning Rational Management

2 students allocated to present cases

Tut 5 Tues 2 Apr

Gastroenterology

Clinical Reasoning Rational Management

2 students allocated to present cases

Tut 6 Tues 9 Apr

Neurology

Clinical Reasoning Rational Management

2 students allocated to present cases

Tut 7 Tues 16 Apr

Endocrinology and Diabetes

Clinical Reasoning Rational Management

2 students allocated to present cases

Tut 8 Tues 23 Apr

Clinical Pathology Common problems 2 students allocated to organise students to focus on each problem.

Tut 9 Tues 28 May

Rheumatology. Paper patients, plus a Quiz.

2 students allocated to organise students to focus on each problem.

Tut 10 Tues 4 Jun

ID (Medicine)

Common problems 2 students allocated to organise students to focus on each problem

Tut 11 11 Jun

Haematology / Oncology

Haematological Cancer: Presentations in the community

2 students allocated to organise students to focus on each problem

Tut 12 Tues 18 Jun

Dermatology

Dermatology treatments, Clinical Reasoning

2 students allocated to present cases

Followed by SL Surgery Topic Format *Tut 13 Week 28th Oct

*ORL Patient-centred presentations

6

Tut 14 Tues 5 Nov

Plastics 1 hr burns and 1 hr flaps and principles of plastic surgery

Tut 15 Tues 12 Nov

ID (Surg) Common problems 2 students allocated to organise students to focus on each problem

For students starting on ‘Surgery’ half year SL Surgery Topic Format Tut 1 Tues 26 Feb

Clinical Methods revision

Tut 13 Tues 5 Mar

ORL Patient-centred presentations

Tut 14 Tues 12 Mar

Plastics 1 hr burns and 1 hr flaps and principles of plastic surgery

Tut 15 Tues 9 April

ID (Surg) Common problems 2 students allocated to organise students to focus on each problem

Then SL Medicine Topic Format Tut 2 Tues 16 Jul

Respiratory

Clinical Reasoning Rational Management

2 students allocated to present cases

Tut 3 Tues 23 Jul

Cardiology

Clinical Reasoning Rational Management

2 students allocated to present cases

Tut 4 Tues 30 Jul

Renal

Clinical Reasoning Rational Management

2 students allocated to present cases

Tut 5 Tues 6 Aug

Gastroenterology

Clinical Reasoning Rational Management

2 students allocated to present cases

Tut 6 Tues 10 Sept

Neurology

Clinical Reasoning Rational Management

2 students allocated to present cases

Tut 7 Tues 17 Sept

Endocrinology and Diabetes

Clinical Reasoning Rational Management

2 students allocated to present cases

Tut 8 Tues 24 Sept

Clinical Pathology Common problems 2 students allocated to organise students to focus on each problem.

Tut 9 Tues 15 Oct

Rheumatology. Paper patients, plus a Quiz.

2 students allocated to organise students to focus on each problem.

Tut 10 Tues 22 Oct

ID (Medicine)

Clinical Reasoning 2 students allocated to organise students to focus on each problem

*Tut 11 Week 28th Oct

*Haematology / Oncology

Haematological Cancer: Presentations in the community

2 students allocated to organise students to focus on each problem

Tut 12 Tues 5 Nov

Dermatology

Dermatology treatments, Clinical Reasoning

2 students allocated to present cases

7

Tutorial 1 Refresher of Clinical Methods

Lead academic advisor: Assoc. Prof Geoff Braatvedt email [email protected]

Resources and revision: North-Nanson Red Book Green handbook from Year 3 Clinical Methods

Background In Years 2 and 3 the students had regular instruction in examination techniques from AProf Andy Wearn in the Clinical Skills Resource Centre. In Year 3 Semester 2, the students took the Professional, Clinical and Communication Skills course (PCCS) with three major components:

• A weekly bedside tutorial in a hospital setting (MMH, NSH, Waitakere or Auckland) for ten weeks. These tutorials followed a syllabus that included history taking, general exam and then CVS, Respiratory, Abdominal and CNS examination techniques. The students had a history taking assessment and an end-of-year clinical ‘must pass’ OSCE. After each tutorial, the students went to see patients on their own. At least three case reports were marked.

• Concurrently, they had a series of whole class lectures that covered history taking (including two sessions where a patient unknown to the lecturer was interviewed in front of the class), how to generate a problem list, ethics, communication skills, etc.

• Small group activities with a tutor covering topics such as history taking, death and dying, ethics, sensitive history taking, clinical reasoning (chest pain, abdo pain and SOB).

Aim for Year 4 Refresher of Clinical Methods In preparation for clinical attachments:

1. revise purpose, structure and documentation of patient history and examination, summary, problem list and differential diagnosis;

2. practice systematic clinical examinations; 3. orientate to writing in admission-to-discharge planners and case notes; 4. Revise resources available and how to get the most out of clinical attachments.

Suggested format • Brief verbal reminder of the ideal structure of interview (Presenting Complaint, Past Medical History,

Medications, Allergies, Family History, Social History, Systems Enquiry), examination, problem list and differential diagnosis. Show how this is documented on local admission-to-discharge forms.(20 min)

• Real patient interview (best with a patient not known to the teacher to make it real and a typical ‘general medical’ type scenario). Followed by discussion. (45 min)

• Demonstration of exam technique (could use actor/patient).(20 min) • Generate a summary and problem list for the patient just seen and document this.(25 min) • Discuss how to write in hospital case notes (SOAP framework) (10 mins)

8

Tutorial 2 Respiratory

Lead academic advisor: Prof Harry Rea email: [email protected]

Resources and revision: Clinical Scenarios as below http://mbchb.auckland.ac.nz/scenarios Phase 1 materials e.g. from Cardiorespiratory course Year 4 Respiratory Teaching from Formal Learning Weeks

First hour – Clinical Reasoning Two nominated students in each cohort will have the tasks of:

• Each finding a case with a respiratory problem, from their ward experiences. Present key features to group (suggest 0-4 Powerpoint slides maximum.).

• Compiling questions from students relating to the cases or respiratory medicine.

Students will need to communicate with each other in order to prepare for the tutorial. After the presentation of the two patient vignettes (max 15 minutes), the remainder of the hour will be spent on:

• discussing relevant aspects of the two cases and covering student queries • clinical reasoning in respiratory medicine:

a. how typical or atypical were the presentations; b. specific history and exam findings, and techniques; c. common differential diagnoses; d. red flags, pitfalls; e. the role and interpretation of common investigations; f. some specific treatments / delivery mechanisms.

• addressing the broader curriculum domains of Hauora Māori and Population Health • covering any other specific topics.

Second Hour - Rational Management

Case 1 (relates to Sudden onset shortness of breath [Resp4]) A 17 year old woman brought into the Emergency Department after becoming suddenly short of breath. She is very distressed and is rushed into the resuscitation room. Her usual medication is inhaled fluticasone 250mcg b.d. and inhaled salbutamol 2 puffs p.r.n. She normally uses her salbutamol once or twice daily. Over the last week she has had increased cough with green sputum, she has been using her salbutamol 6-8 times a day and she has been waking at night. By the time she arrives at hospital she is breathless at rest.

Questions 1. How do you assess the severity of her asthma? 2. What are your initial investigations? 3. What is your initial treatment (specify the dose and frequency of medicines that you would use)? 4. If she does not respond to the initial treatment, what would you do next?

If she were regularly using salbutamol 6-8 times a day, 5. What factors might be contributing? 6. How could you improve her management?

Case 2 (relates to Chronic shortness of breath [Resp2]) A 74 year old woman presents to the Emergency Department with worsening shortness of breath. She is well known to the department, needing hospital admissions around two to three times per year. She says that she is always short of breath and has difficulty getting to the shops but things are currently much worse than usually. She says she is breathless just lying on the bed. Her usual medication is inhaled salbutamol 2 puffs prn and theophylline (Nuelin SR) 250mg twice daily. On examination she has a temperature of 37.8°C and a respiratory rate of 28 per minute. Pulse oximetry (while breathing air) shows a saturation of 87%.

Questions 1. Are there any other investigations that you would request?

9

2. How would you manage her exacerbation of COPD? After she recovers from her exacerbation she is reviewed by her GP. She wants to know whether she should be treated with inhaled fluticasone (her son is on treatment with fluticasone for asthma).

3. How would you respond to her question? 4. Are there any other treatments (either pharmacological or non-pharmacological) that you should

consider for the ongoing management of her COPD?

Case 3 (relates to Pneumonia [Resp5]) You visit a 98 year old woman in a private hospital where she has been a resident for the past three years. In the past she has had a stroke that has affected her ability to swallow. She has a fever of 37.8°c and is coughing up purulent sputum. On examination she has poor dentition, crackles throughout the left side of her chest posteriorly and dullness and decreased air entry at the left base. You send her to the local Emergency Department. A chest X-Ray at the Emergency Department shows left upper lobe consolidation and large left pleural effusion.

1. What is the likely diagnosis? 2. You aspirate some pleural fluid. What tests do you request on the pleural fluid?

After aspirating the pleural fluid you commence her on treatment. 3. What is your initial treatment? 4. How would the results of your investigations influence your management?

You successfully treat the pleural effusion and she initially shows a marginal clinical improvement (but never complete apyrexia). However, over the next few days her fever gradually worsens, the cough becomes productive, and her lung function deteriorates. Chest CT scan reveals spread of the existing pneumonia and development of multiple cavitating lesions with air-fluid levels.

5. What is the likely diagnosis and how would you manage her? 6. If the CT showed persistent pneumonia but no cavitating lesions, what would be your approach to

management?

Case 4 (relates to Deep vein thrombosis [Blood3] and Sudden onset shortness of breath [Resp4]) A previously fit 30 year woman presents with sudden onset of breathlessness and pleuritic chest pain. She is a non-smoker, regularly exercises and has not previously been limited by breathlessness. She is on treatment with the combined oral contraceptive but takes no other medication. Two weeks ago she flew from Los Angeles to Auckland. On examination she is afebrile. Her blood pressure is 120/70, her heat rate is 105 per minute and her respiratory rate is 24 per minute. On auscultation, her chest is clear. Her left calf is swollen. A CXR shows no abnormality.

Questions 1. What is the differential diagnosis? 2. What other investigations would you request? 3. If the suspected diagnosis is confirmed, how would you treat her? 4. Is there any other advice you should give her?

Additional Cases These may be covered if sufficient time or as SDL.

Case 5 (relates to Haemoptysis and abnormal chest X-ray [Resp6]) A 65 year old man returns to the GP clinic where you are on your attachment. He has had a prolonged cough and the GP arranged a chest X-Ray; this has shown a left hilar mass. He has come back for the result and reports that he coughed up some blood recently. He is a long term smoker. Clinically he is clubbed. Laboratory investigations show only mild hypercalcaemia. You suspect he has a primary bronchial carcinoma.

Questions 1. What tests would you consider in order to obtain a definite pathologic diagnosis? 2. What additional tests would you routinely undertake in order to determine the patient’s suitability for

surgical resection? 3. What is the possible significance of the hypercalcaemia?

Six months later the patient re-presents with a one month history of worsening lower thoracic back pain. Radiology shows a lytic lesion in T12 consistent with a metastasis.

4. What would be your management of this problem?

10

5. List community-based services that you might now enlist to assist in management of this man in his own home.

Case 6 (relates to Worsening shortness of breath [Resp8]) A 67 year old man presents with slowly progressive shortness of breath on exertion. He has a history of working in mines in Western Australia during his 20s and 30s but has subsequently worked in office jobs. His chest radiograph shows evidence of diffuse lung disease (consistent with pulmonary fibrosis).

Questions 1. What findings do you expect on auscultation? 2. With regards to aetiology of his lung disease, what other points in the history would you explore and

why? He develops oxygen desaturation on exercise.

3. Describe the likely pathophysiologic mechanism. 4. Describe the likely pattern of abnormality on spirometry. 5. Describe the likely abnormalities on arterial blood gases.

Case 7 (relates to Daytime sleepiness [Resp9]) A 52 year old man comes to your clinic due to worsening snoring, sleep fragmentation, unrefreshing sleep and excessive daytime sleepiness. He says he spends 9 hours a night in bed and sleeps for at least 7 hours a night. He says he occasionally wakes gasping for breath, and he gets up 2 or 3 times a night to pass urine. He complains of morning headache. He scores 18/24 on the Epworth Sleepiness scale (upper limit of normal 9/24) and this is causing problems at his work. His partner says that he frequently stops breathing and thrashes around during sleep. His past medical history includes current diagnoses of Type 2 diabetes, gout, hypertension, dyslipidaemia and he is on appropriate drug therapy. He denies symptoms of angina, TIA or stroke. Physical examination reveals a tired-looking but not overtly sleepy, morbidly obese man with a BMI of 48. The neck is short with increased circumference. The nose is patent. On examination of the oral cavity the tongue is large (Malampati score is 4), tonsils are symmetrically mildly enlarged (Grade 1/4), and the oropharyngeal structures appear generally erythematous and swollen. Blood pressure is 130/85, Pulse 88 bpm regular, SpO2 95%. Cardiovascular, Respiratory and Neurological examinations are otherwise normal. He proceeds to a home overnight oximetry study. A 7 hour recording looks to be of good quality on manual review. It shows multiple discrete dips in oxygen saturation occurring throughout the recording period at a rate of 62/hour (for >4% dips). The average SpO2 overnight is 90.1%, lowest is 63%. Average desaturation is 13.9%.

Questions 1. What is the likely diagnosis? 2. What is the best treatment for this man at this time, and why? 3. What other treatments should or could be advised?

11

Tutorial 3 Cardiology

Lead academic advisor: Prof Rob Doughty email [email protected]

Resources and revision: Clinical Scenarios as below http://mbchb.auckland.ac.nz/scenarios Phase 1 materials e.g. from Cardiorespiratory course Year 4 Cardiology Teaching from Formal Learning Weeks

First hour – Clinical Reasoning Two nominated students in each cohort will have the tasks of:

• Each finding a case with a cardiac problem, from their ward experiences. Present key features to group (suggest 0-4 Powerpoint slides max.).

• Compiling questions from students relating to the cases or cardiology.

Students will need to communicate with each other in order to prepare for the tutorial. After the presentation of the two patient vignettes (max 15 minutes), the remainder of the hour will be spent on:

• discussing relevant aspects of the two cases and covering student queries • clinical reasoning in cardiology:

a. how typical or atypical were the presentations; b. specific history and exam findings, and techniques; c. common differential diagnoses; d. red flags, pitfalls; e. the role and interpretation of common investigations; f. some specific treatments

• addressing the broader curriculum domains of Hauora Māori and Population Health • covering any other specific topics.

Second Hour - Rational Management

Case 1 (relates to Valvular heart disease [CVS5]) A 38 woman attends her GP's clinic because she is getting very short of breath, even at rest. She is 28 weeks pregnant, and is on no treatment. She denies chest pain but has noticed palpitations. On examination her heart rate is 90 per minute with an irregular rhythm, and the GP hears a murmur. There are bi-basal inspiratory crackles and mild pitting ankle oedema. She is reviewed urgently by the Maternal Medicine registrar and an ECG shows an irregularly irregular rhythm. An echocardiogram shows an enlarged left atrium, thickened mitral valve leaflets with restricted opening, and normal left ventricular systolic function.

Questions 1. What is the diagnosis and why is she breathless? 2. How would you treat her breathlessness? 3. What needs to be considered during labour?

Case 2 (relates to Acute chest pain [CVS2]) A 75 year old man presents to the ED with central chest pain. It has been present for the past 40 minutes and came on while he was resting. He is feeling short of breath, sweaty and nauseous. On examination he is pale, cool, and sweaty. His pulse feels weak and you estimate it at 100bpm. His BP is 170/100 on arrival in the ED. There is a 4th heart sound. His ECG shows ST-segment elevation in leads II, III, and aVF and ST segment depression in other leads.

Questions 1. What is the diagnosis? 2. What is your initial management of this patient? 3. What else should you consider doing to treat this man’s problem, thinking here about intervention? 4. How would you manage him in the long term?

12

Case 3 (relates to Arrhythmias [CVS6]) A 64 year old man is brought to the Emergency Department by ambulance. He had collapsed, temporarily lost consciousness, and was observed to become pale at the time of collapse. When the ambulance crew arrived, it was noted that his pulse rate was very rapid. On examination his apex beat is not displaced and you cannot hear any added heart sounds or murmurs, but you are not entirely confident about this because the heart rate is 150bpm. His BP is 140/80.

Questions 1. What are the possible causes of his collapse? 2. What is the most likely explanation? 3. What investigations would you request? 4. What is your initial treatment? 5. Are there any other treatment options you should consider?

Case 4 (relates to Secondary hypertension [Endo7]) A 35 year old woman visits her GP as she wants to start on the oral contraceptive pill. You are asked to take her blood pressure and it is 180/105. She has lifelong asthma and uses her salbutamol inhaler frequently. At 32 years of age she presented with headaches and a consistent BP of 170/110 to 180/110. She has been on enalapril 20mg/day ever since, and has tolerated it well. You find her BP to be 160/105 and her notes indicate that it has never been better than this, despite therapy. Her clinical examination is normal. Serum creatinine is 90micromol/l and her serum K+ is 4.0mmol/l.

Questions 1. Would you do further investigations? If so, what? 2. How would you gain better control of her BP through non-pharmaceutical and pharmaceutical

means? 3. What would you do if she told you she wanted to become pregnant?

Case 5 (relates to Stable angina [CVS4]) You are on your GP placement and you are asked to see a 69 year old man who suffers from hypertension. He complains of chest pain when he walks and comments that it is particularly bad when he walks up hills or stairs. It gets better when he rests. For the last three days he has been having pain at rest. He is on treatment with atenolol 50mg and a GTN spray as required. He has a 20 pack year smoking history. On examination his pulse rate is 60 per minute and his BP is 170/100. The rest of the examination is normal. An ECG during pain shows 1 - 2mm of ST segment depression leads V1 - V4. These changes resolve following treatment with GTN. His Troponin T is 0.10µg/l (<0.01). A random serum cholesterol is 7.0mmol/l.

Questions 1. What is the diagnosis? 2. What is your initial management of this patient? 3. What else should you consider doing to treat this man’s problem? 4. How would you manage his cardiovascular risk factors?

13

Tutorial 4 Renal Medicine

Lead academic advisor: AProf Helen Pilmore email: [email protected]

Resources and revision: Clinical Scenarios as below http://mbchb.auckland.ac.nz/scenarios Phase 1 materials e.g. from Genitourinary course Year 4 Renal Teaching from Formal Learning Weeks

First hour – Clinical Reasoning Two nominated students in each cohort will have the tasks of:

• Each finding a case from their ward experience – students should liaise with each other and make sure the cases are as below

One student focusing on Acute Kidney Injury One student focusing on Chronic Kidney Disease

• Present key features to group (suggest 0-4 Powerpoint slides max.).

Students will need to communicate with each other in order to prepare for the tutorial. After the presentation of the two patient vignettes (max 15 minutes), the remainder of the hour will be spent on:

• discussing relevant aspects of the two cases and covering student queries • clinical reasoning in that specialty:

a. how typical or atypical were the presentations; b. specific history and exam findings, and techniques; c. common differential diagnoses; d. red flags, pitfalls; e. the role and interpretation of common investigations; f. some specific treatments: dialysis and medical treatment.

• addressing the broader curriculum domains of Hauora Māori and Population Health • covering any other specific topics.

Second Hour - Rational Management

Case 1 (relates to Chronic kidney disease / asymptomatic worsening renal function [GU7]) Male, aged 23 years History He was well until about 2 years ago when he began to notice some tiredness and lethargy. Six months later he was noted to have proteinuria and was slightly anaemic with a haemoglobin of 104g/l. Four months before admission he was found to have moderately severe hypertension (BP 200/120). This was only treated sporadically with atenolol and a thiazide as the patient did not appreciate the importance of treatment. There is no past history of renal disease although he has had recurrent sore throats over the years. He is single and works as an office worker in a timber firm. Examination He was a rather thin, pale young man CVS: BP 170/125 lying and 165/130 sitting Pulse 88 and regular The cardiac apex was in the anterior axillary line and there was a marked left ventricular lift. There

was a prominent fourth heart sound, but there were no murmurs and there was no evidence of cardiac failure

Fundi: There were Grade 3 retinal changes with several flare haemorrhages in the vicinity of the disc Peripheral pulses: All palpable and no femoral delay. No renal bruits. RS: The chest was clinically clear AS: The abdomen was soft and non-tender and there were not palpable masses. The kidneys were not

palpable CNS: There were no focal neurological signs Investigations Biochemistry: urea 34mmol/l (N: 3.2 - 7.7) creatinine 500µmol/l (N: 90 -120)

14

eGFR 13ml/min sodium 136mmol/l potassium 6.5mmol/l HC03

- 15mmol/l calcium 1.95mmol/l (N: 2.10 - 2.48) uric acid 0.65mmol/l (N: 0.25 - 0.51) phosphate 2.2mmol/l (N: 0.9-1.6) cholesterol 6.1mmol/l (N: 3.0 - 5.2) total protein 66g/l (N: 62-80) albumin 38g/l (N: 35-47) globulin 28g/l (N: 25-35) Other liver function tests normal 24 hr urinary protein: 3 grams Protein/creatinine Ratio: 430mg/µmol (N:<20) Haematology: Hb 81g/l ESR 45mm/hr Mid-stream urine: Two urines showed microscopic haematuria (50 RBC/cmm) There were occasional granular casts There was no growth on culture Chest X-Ray: No pulmonary lesion was seen. The heart shadow was moderately enlarged. Films of skull and hands showed evidence of secondary hyperparathyroidism Ultrasound of kidneys: Right kidney length 8.5cms Left kidney 8.8cms Comment: small kidneys with echogenic parenchyma and some increase in peripelvic fat. No

signs of dilatation of the pelvis or the calyces ECG: Sinus rhythm at 80/minute. Evidence of left ventricular hypertrophy

Questions 1. What is the likely diagnosis and on what evidence? 2. What is the differential diagnosis? 3. The uric acid is raised. Does this mean the patient has gout? 4. Why is he anaemic and what treatment would you recommend? 5. What treatment would you recommend for his hypertension? 6. What are the consequences of his low calcium and high phosphate? How would you manage this?

Case 2 (relates to A child with generalized swelling [Paed9] and SLE / fatigue / pain and swelling in the hands [Rh3]) History: Miss NS age 18 developed severe ankle swelling over 3 days. In the morning her face and eyelids were swollen. During the day she passed very little urine but at night had to rise 3 times to pass quite large volumes of urine. She did not have nocturia in the past. She was tired but otherwise well and on no treatment. Examination: blood pressure 105/85 pulse 72 and regular, JVP-1cm heart sounds normal, no gallop rhythm pitting oedema to knees distended abdomen with shifting dullness bilateral small pleural effusions Investigations: Biochemistry: urea 5.1mmol/l creatinine 70umol/l reatinine clearance 103ml/min eGFR >60 mls/min potassium 4.2mmol/l sodium 144mmol/l albumin19g/l Haematology: normal MSU: WBC 0 RBC 15

15

Epithelial cells 1 protein 3+ glucose-ve no growth PCR: 654 mg/mmol Serology: ASO titres 1-100 Antinuclear factor 1:40 Anti ds DNA – negative Complement C3 and C4 – normal

Questions 1. What clinical syndrome does this patient have? 2. How would you manage her in terms of diagnosis and treatment? 3. What is the differential diagnosis? 4. What complications might she have?

Case 3 (relates to Acute kidney injury [GU 02] and Postoperative complications [Abdo 22]) A 40 year old diabetic man has a laparotomy. Preoperatively his serum creatinine was 80 micromol/l, eGFR 70ml/min. Postoperatively he receives diclofenac and morphine as pain relief. You are called to see him because of nausea and vomiting. His abdomen is soft and non-tender with normal bowel sounds. His JVP is -2cm. A serum creatinine is 750micromol/l.

Questions 1. Why has he developed renal failure? 2. What were his risk factors for developing this problem? 3. Do you have any concerns about the use of morphine in the presence of his renal failure?

Case 4 (relates to Acute kidney injury [GU2], Chronic kidney disease / asymptomatic worsening renal function [GU7], and Heart failure [CVS3]) History: A 75 year old male develops a high fever, vomiting and abdominal pain. He noted that his urine volume had decreased and routine biochemical investigations carried out by his general practitioner showed a urea of 27mmol/l, creatinine 340micromol/l and a potassium of 6.8mmol/l. He is admitted to hospital and given a fluid load with normal saline. Two hours after the completion of the saline he becomes severely breathless and is hypoxic.

Questions 1. Describe your approach to the diagnosis of this patient and his problems. Areas to be addressed

should include the investigations necessary in making a diagnosis (including organ imaging, urine investigations and other tests).

2. Describe how you would manage the patient.

Additional Cases These may be covered if sufficient time or as SDL.

Case 5 (relates to Epilepsy / seizure [N6] and Accidental overdose [ED6]) A 25 year old man has epilepsy, which is well controlled on treatment with phenytoin 300 mg per day. He develops nephrotic syndrome. His serum albumin is 20 g/l. A phenytoin level is checked and is found to be 20 µmol/l (therapeutic range = 40 – 80 µmol/l). His dose of phenytoin is increased to 360 mg per day and he develops drowsiness and ataxia.

Question 1. Why?

Case 6 (relates to Chronic kidney disease / asymptomatic worsening renal function [GU7]) A 75 year old woman has hypertension and is on treatment with Amizide (hydrochlorothiazide and amiloride). Her serum creatinine is 180micromol/l and her serum potassium is 4.5mmol/l. Her blood pressure remains elevated at 170/105 and she is started on treatment with quinapril 20 mg twice daily. Two weeks later her serum potassium is 6.0mmol/l.

16

Questions: 1. Why is her potassium elevated? 2. What do you think her eGFR would be?

How would you manage her hyperkalaemia?

17

Tutorial 5 Gastroenterology

Lead academic advisor: Assoc Prof Alan Fraser email [email protected] Dr David Rowbotham email [email protected]

Resources and revision: Clinical Scenarios as below http://mbchb.auckland.ac.nz/scenarios Phase 1 materials e.g. from GI course Year 4 Gastroenterology and Hepatology Teaching from Formal Learning Weeks

First hour – Clinical Reasoning Two nominated students in each cohort will have the tasks of:

• Each finding a case with a gastroenterological problem, from their ward experiences, Present key features to group (suggest 0-4 Powerpoint slides max.).

• Compiling questions from students relating to the cases or gastroenterology.

Students will need to communicate with each other in order to prepare for the tutorial. After the presentation of the two patient vignettes (max 15 minutes), the remainder of the hour will be spent on:

• discussing relevant aspects of the two cases and covering student queries • clinical reasoning in gastroenterology:

a. how typical or atypical were the presentations; b. specific history and exam findings, and techniques; c. common differential diagnoses; d. red flags, pitfalls; e. the role and interpretation of common investigations; f. some specific treatments / delivery mechanisms.

• addressing the broader curriculum domains of Hauora Māori and Population Health • covering any other specific topics.

Second Hour - Rational Management

Case 1 (relates to Chronic bowel motility problems [Abdo10]) A 37 year old male presents with right iliac fossa discomfort over the past 2 months. He also has central abdominal cramping pain 30 minutes after every meal but particularly larger meals. There is sensation of distension when he has the pain. Usually the pain lasts for 1-2 hours. There is tenderness on deep palpation in the right iliac fossa. Blood tests show a mild anaemia (MCV normal), elevated platelet count and a raised CRP

Questions 1. Give a physiological explanation for the two types of pain? 2. What is the most likely diagnosis and what investigation is appropriate

The symptoms improve with treatment. 3. What medication could be helpful?

The medication is helpful but 4 years later he presents with similar symptoms that do not respond to medication.

4. Why would the medication not be effective now? He has an operation that involves removal of 100 cm of terminal ileum.

5. What are two important consequences of resection of the terminal ileum? He is given cholestyramine and this improves his symptoms.

6. What symptom will improve and why does this drug help? He has further surgery for Crohn’s disease – this involved removal of more small bowel – approx 100cms. On recovery from the operation he has high volume diarrhea - over 2 litres per day. He is started on TPN (intravenous nutrition because of rapid weight loss.

7. What is the reason for the high volume diarrhea? 8. What problems may occur delivering digestive products directly into systemic circulation?

To replace normal gut function protein is given as amino acids. 9. Describe the normal process of protein digestion that leads to breakdown of protein to amino acids?

18

10. What is the advantage of retaining the colon and ileo-caecal valve? 11. When introduction of oral feeding is attempted why is it an advantage for this to be iso-osmolar.

Why? 12. How can net intestinal secretion be reduced? 13. How can increased small bowel absorption be encouraged?

After 6 months he is able to take enough calories orally to maintain adequate nutrition and the TPN can be discontinued.

14. What process is happening to enable oral intake? 15. What GI hormone(s) is trophic for the small bowel?

Case 2 (relates to Adult with abdominal mass [Abdo 5], and Anaemia [Blood4]) A 65 year old male presents with tiredness and breathless on exertion. He is found to have iron deficiency anaemia - Hb 72 g/L and MCV 71. On direct questioning there were no other relevant symptoms. He has not observed any blood in the bowel motions. His diet was normal – low dietary iron was not considered to be a problem.

Questions 1. He has no symptoms of indigestion and no bowel symptoms. The first investigation chosen was a

colonoscopy. Explain the rationale for this decision? Colonoscopy shows a carcinoma of the caecum. He has a right hemicolectomy.

2. What is the blood supply of the resected colon? Presentation with colonic obstruction is unusual for cancer of the caecum (except in one circumstance) but is common for cancer involving the sigmoid than cancer involving the caecum. 3. Why is this?

His brother has a colonoscopy in view of this diagnosis. To perform a colonoscopy the bowel needs to be clean. If this was achieved by drinking saline then this would require over 10 litres of fluid and a net absorption of 4 litres of water (and sodium).

4. In what situation would this be dangerous for the patient? A washout solution has been designed that requires only 4 litres of fluid with no net absorption of water or electrolytes.

5. How is this achieved? Colonoscopy technique depends on being able to straighten loops as they develop. The technique depends on knowing which parts of the colon are fixed and what areas are prone to looping.

6. What are these areas? What is meant by being “fixed”? He is found to have colonic polyps. These are removed at the time of colonoscopy.

7. What features of the polyp(s) would suggest that there is a high risk of further polyps that could lead to colon cancer?

Further inquiry reveals that the father of these two brothers also had bowel cancer diagnosed at the age of 49 years and the grandfather had bowel cancer at age of 65 years.

8. What is the likely diagnosis? 9. What should be the management of the children of these two brothers? 10. If this 65 year old man had a normal colonoscopy - what would be the approach to his problem?

Examination of the oesophagus, stomach and duodenum was normal - biopsies normal. 11. What may be an appropriate next investigation?

A study of the small bowel using a “capsule camera”. This is camera that is swallowed and moves by normal gut peristalsis. It sends pictures to a data recorder every second. The patient is fasted and has taken some fluid the night before to flush out most of the bowel contents. The capsule is swallowed at 8am (having missed breakfast). A meal is allowed after 3 hours. The battery life of the capsule camera is 9 hours.

12. How long will the capsule spend in the oesophagus?. How many pictures of the oesophagus? 13. How long will the capsule be in the stomach? What is the rate of gastric emptying for a solid meal? 14. How long is the small bowel? How long will the capsule be in the small bowel? Will the capsule

reach the colon before the battery runs out? The test is performed during fasting.

15. What is the important motility pattern during fasting that will propel the capsule through the small bowel? What hormone is thought to involved in this motility pattern? Where does it start from?

16. What would be the limitations of capsule endoscopy in the colon? 17. What are some potential findings at capsule endoscopy?

19

Case 3 (relates to Chronic upper abdominal symptoms [Abdo12]) A 55 year man presents with heartburn (retrosternal burning discomfort occurring mainly after meals) and also mild epigastric discomfort. He has tended to self-medicate with antacids. He smokes 10 cigarettes per day. He drinks 10-15 units of alcohol per week. His weight has gradually increased over the past 5 years, mostly with central obesity. He has some variable bowel habit with a predominance of diarrhoea. He has intermittent lower abdominal cramping pain prior to defaecation. This has been his normal bowel habit for many years. There is some aggravation of bowel symptoms (and also reflux-type symptoms) with fatty foods and alcohol.

Questions 1. What are the likely diagnoses? 2. What lifestyle advice could you suggest? 3. What would be your advice regarding appropriate medication? 4. What investigation(s) would be appropriate? Are there issues with timing of investigation and starting

medication? 5. His GP requested a serology test for Helicobacter infection – this was positive. What is your advice

regarding treatment for Helicobacter? How likely is treatment going to help his symptoms? Gastroscopy showed Grade III reflux oesophagitis. 6. Does this change your recommended treatment and for how long? 7. Are there any long-term side-effects of treatment? 8. What dose change would be appropriate if initial treatment is not effective

Case 4 (relates to Chronic liver disease [Abdo15]) A 52 year old man with a history of heavy alcohol intake presents with jaundice, abdominal distension, fever and increasing drowsiness. On examination he has a flapping tremor and diffuse abdominal tenderness. He is unclear what medicines he is taking. Blood tests show abnormal liver enzyme tests, a low serum albumin at 25g/L and a prothrombin ratio of 2.0.

Questions 1. What is the most likely explanation for his drowsiness? 2. What are the potential precipitating factors? 3. What investigations would you do? 4. How would you manage this patient?

Additional Cases These can be covered if sufficient time or as SDL.

Case 5 (relates to Upper gastrointestinal bleeding [Abdo7]) A 70 year old man presents with haematemesis and melaena. He was hypotensive on arrival (also with a tachycardia > 110/min) but now has stable BP (after resuscitation). He has a history of a previous MI (complicated by heart failure) and takes frusemide 40mg mane and cilazapril 5mg mane. He has had a long history of back and hip pain. Hb is 80g/l. Gastroscopy shows a visible vessel in the base of a gastric ulcer (1cm).

Questions 1. What are the possible causes of his gastric ulcer? 2. What is the relevance of his back and hip pain? 3. What is his risk of re-bleeding and in-hospital mortality risk? 4. What steps can be taken to prevent rebleeding in hospital and later recurrence of the bleeding post

discharge?

Case 6 A 35 year old woman has lower abdominal discomfort, nausea and abdominal distension. Her normal bowel habit is to pass one motion every 7 days (long-standing symptoms since teenage years). Fibre supplements were not effective and lactulose causes excess abdominal bloating. She has been able to get some relief with the use of laxatives containing Senna but is now concerned that the dose required for effect is increasing. On examination she has an easily palpable and loaded sigmoid colon (and also caecum). Sigmoidoscopy shows a pigmented appearance of the rectal mucosa

20

Questions 1. What investigations would be appropriate? 2. Are there any potential disadvantages with Senna? 3. What alternatives are there to treat the constipation? 4. Why does lactulose cause bloating and excess flatulence?

Case 7 (relates to Chronic liver disease [Abdo5]) A 38 year old female presents with tiredness. Routine blood tests have shown a mild elevation in ALT and AST (75-125 U/L) noted on serial blood tests over the last 6 months. On further testing the Hep Bs Ag is negative; Hep Bs antibody positive. Hepatitis C Antibody is positive. ANA antibody is positive 1:32 and smooth muscle antibody is negative.

Questions 1. What other history and investigation is relevant? 2. What treatment may be needed (depending on results of investigation)? 3. Does she have auto-immune hepatitis?

21

Tutorial 6 Neurology

Lead academic advisor: Prof Alan Barber email: [email protected]

Resources and revision: Clinical Scenarios as below http://mbchb.auckland.ac.nz/scenarios Phase 1 materials e.g. from Neurosciences course Year 4 Neurology Teaching from Formal Learning Weeks

First hour – Clinical Reasoning Two nominated students in each cohort will have the tasks of:

• Each finding a case with a neurological problem, from their ward experiences. Present key features to group (suggest 0-4 Powerpoint slides max.).

• Compiling questions from students relating to the cases or neurology.

Students will need to communicate with each other in order to prepare for the tutorial. After the presentation of the two patient vignettes (max 15 minutes), the remainder of the hour will be spent on:

• discussing relevant aspects of the two cases and covering student queries • clinical reasoning in neurology:

a. how typical or atypical were the presentations; b. specific history and exam findings, and techniques; c. common differential diagnoses; d. red flags, pitfalls; e. the role and interpretation of common investigations; f. some specific treatments / delivery mechanisms.

• addressing the broader curriculum domains of Hauora Māori and Population Health • covering any other specific topics.

Second Hour - Rational Management

Case 1 (relates to Epilepsy / seizure [N6]) A 20 year old woman is brought to the Emergency Department after having an apparent convulsive episode at a bar.

Questions 1. What are the differential diagnoses for this episode and what measures would you take to help

distinguish between these? 2. What investigations would you obtain and why? 3. What lifestyle advice would you give her?

She has a second convulsive episode one month later and you decide to start treatment for epilepsy. 4. What anti-convulsant medications might you consider and why?

She comes to see you four years later to tell you that she wishes to become pregnant. She wants to know if anticonvulsant medications have any effect on the foetus.

5. What do you tell her? She goes on to have a healthy baby but stops her anticonvulsant medication after delivery because of concerns surrounding breast feeding. She is brought into the emergency department by ambulance with seizure activity that has been going on for more than 30 minutes.

6. How do you manage this woman a. In the emergency department? b. In the longer term?

Case 2 (relates to Stroke / TIA / sudden onset unilateral weakness [N7]) A 55 year old man collapses at work with a sudden onset of right sided weakness. His wife tells you that recently he has had short episodes when he couldn't find the right words. He has fairly well controlled hypertension and has been a lifelong smoker.

22

Questions 1. How would you investigate this man

a. In the emergency department? b. In the stroke unit?

2. How would you treat this man acutely a. If brain imaging showed evidence of cerebral infarction? b. If brain imaging showed evidence of intracerebral hemorrhage?

3. What measures would you take to reduce the risk of further vascular events a. In the emergency department? b. In the stroke unit? c. What lifestyle measures would you advise him to make?

Case 3 (relates to Headache [N3]) A 50 year old woman presents to her GP with daily frontal headaches for three months. When her headache is at its worst her vision seems blurred. There have been no other neurological symptoms. She has been taking paracetamol and ibuprofen on a daily basis for the past three months.

Questions 1. What are the differential diagnoses of this woman’s headaches and why? 2. How would you investigate this woman and why? 3. Regarding therapy:

a. How would you treat her the acute phase? b. How would you treat her to prevent further headaches? c. What lifestyle measures would you advise her to make?

Case 4 (relates to Tremor and parkinsonism [N9]) A 68 year old man presents complaining of trembling in his right hand progressing over the last 2 years. His wife has noticed that his writing has got worse and that he is "slowing down" especially when getting up from his chair. Past medical history includes GORD and hypertension, for which he takes omeprazole, metoclopramide and bendrofluazide.

Questions 1. What are the differential diagnoses for this man’s condition? 2. How would you investigate him? 3. What are the treatment options for each of the differential diagnoses that you have provided?

Case 5 (relates to Neuromuscular disease / generalised muscle weakness [N10]) A 24 year old woman presents with a four day history of reduced vision in her left eye. She has noticed that it is uncomfortable to move this eye. She is previously fit and well. You make a diagnosis of optic neuritis.

Questions 1. How would you investigate this woman and why? 2. What therapy would you consider?

She makes a good recovery with a return in her visual acuity to near normal but she is left with a pale optic disc. However, six months later she presents with bilateral lower limb weakness, sensory loss distal to the umbilicus and urinary urgency that has come on over a few days.

3. What do you think is going on? 4. What therapy would you consider?

Over the next 3 months there is some improvement in her function and she can mobilise independently but she needs some help with her activities of daily living.

5. What treatment options are available?

23

Tutorial 7 Endocrinology

Lead academic advisor: Assoc Prof Andrew Grey email [email protected]

Resources and revision: Clinical Scenarios as below http://mbchb.auckland.ac.nz/scenarios Phase 1 materials e.g. from Reproduction and Development course Year 4 Endocrinology and Diabetes Teaching from Formal Learning Weeks

First hour - Clinical Reasoning Two nominated students in each cohort will be pre-allocated the tasks of:

• Each finding a case with an endocrine problem, from their ward experiences. Present key features to group (suggest 0-4 Powerpoint slides max.).

• Compiling questions from students relating to the cases or endocrinology.

Students will need to communicate with each other in order to prepare for the tutorial. After the presentation of the two patient vignettes (max 15 minutes), the next 15 minutes to be spent on:

• discussing relevant aspects of the two cases and covering student queries • clinical reasoning in endocrinology:

a. how typical or atypical were the presentations; b. specific history and exam findings, and techniques; c. common differential diagnoses; d. red flags, pitfalls; e. the role and interpretation of common investigations; f. some specific treatments / delivery mechanisms.

• you should restrict student cases and discussions to 30 min in order to cover cases below in the first hour.

Type 1 Diabetes Mellitus

Case1 (relates to Acute presentation of diabetes mellitus [Endo2]) A 17 year old European male presents with a short history of exhaustion, weight loss (6kg), excessive thirst, polyuria and nocturia. Over the 24 hours prior to admission he has become increasingly drowsy and, on the morning of his admission, his parents had difficulty waking him from sleep. Vomitus was on the pillow. Examination : BP 65/40 pulse 100 JVP very low skin turgor reduced Kussmaul

respiration Biochemistry : sodium 142 mmol/l pH 6.91 potassium 6.1mmol/l glucose 36.2mmol/l creatinine 210 µmol/l urine ketones +++ HCO3 3

Questions 1. What is the diagnosis? 2. What would your management be with regard to:

a. fluid and electrolytes? b. insulin?

3. Should the patient have: a. naso-gastric tube? b. bladder catheter? c. CVP line?

4. What clinical and biochemical measures would you follow during his initial treatment period? 5. How frequently would you test them? 6. When and how would you change the patient to subcutaneous insulin?

24

Case 2 (relates to Type 1 diabetes mellitus [Endo10]) A 25 year old woman with type 1 diabetes of 19 years duration is admitted to hospital following a motor vehicle accident. She is unconscious but has no obvious injuries and no focal neurological signs. Her blood sugar is 0.9mmol/l. A diagnosis of insulin-induced hypoglycaemia is made.

Questions 1. What are the important counter-regulation hormones to hypoglycaemia? 2. Describe your immediate management of this patient. 3. When she recovers, what information would you like to know to help explain this episode and avoid

others? 4. What is hypoglycaemic unawareness? 5. What advice would you give the patient and her partner on the management of hypoglycaemia at

home when this is: a. mild ? b. severe?

Type 2 Diabetes Mellitus

Case 3 (relates to Type 2 diabetes mellitus [Endo11]) A 65 year old man with known type 2 diabetes for 15 years consults you about his disease. Despite keeping as much as possible to a “sensible” diet and walking twice weekly, his HbA1c remains at 74 mmol/mol. Home glucose recordings are often in double figures, and he generally feels a bit tired. Current medication is metformin 850mg bd and gliclazide 160mg bd. He can’t afford sitagliptin and has not been prescribed pioglitazone before.

Questions 1. What is HbA1c? 2. What are the mechanism of actions of thiazolidinediones and sitagliptin? 3. How might you start him on insulin?

a. What type of insulin? b. What dose? c. At what time of the day?

4. What problems may he encounter with insulin use?

Case 4 (relates to Acute presentation of diabetes mellitus [Endo2], Type 2 diabetes mellitus [Endo11] and Unexplained loss of consciousness [ED3]) A 72 year old man presents with a two week history of excessive thirst, polyuria and nocturia. He had been told by his GP that he had “borderline diabetes” some five years earlier. His intense thirst he has been quenching with “Coca Cola”, recently about 3 litres a day. On the evening prior to admission he felt very sleepy and went to bed early. Next morning his wife found him difficult to wake and somewhat incoherent. Examination : BP 110/60 pulse 95 JVP very low skin turgor reduced tongue dry Biochemistry: sodium 165 pH 7.38 potassium 5.7 glucose 72.4 creatinine 230 urine ketones + HCO3 19

Questions 1. What is the diagnosis? 2. How much glucose is there in Coca Cola? 3. In what way would the management differ from that in diabetic ketoacidosis? 4. Assuming the patient has Type 2 diabetes, when and how would you take the patient off insulin?

25

Second Hour - Rational Management

Case 1 (relates to Hip fracture [M21]) A 70 year old woman who is normally fit and well sustained a forearm fracture 2 months ago she after falling from a standing height. Her mother had a hip fracture at age 85, and she is concerned that she may have osteoporosis.

Questions 1. What are the risk factors for fragility fracture?

You request bone densitometry and this shows a T score of –3 at the hip. 2. What is a T score? 3. Are there any other investigations that you need to order?

Her BMI is 22kg/m2. 4. What is her absolute risk of fracture in the next 5 years? 5. What treatment options could you offer her? How effective are they? 6. What adverse effects could occur with these treatments and what advice would you give her about

how to take these medicines?

Case 2 (relates to Adrenal crisis [Endo5]) A 45 year old Afghani man presents to the Emergency Department feeling very unwell. He tells you that for several months he has been feeling dizzy after standing up. He has lost 12 kg in weight over the last 1-2 years. He also says he has been feeling tired. A cousin who has been away in Europe for a year has commented that his skin is becoming darker. On examination he is very thin and has a tanned complexion. His lying BP is 95/60 and falls to 70/50 on standing. His JVP is 0cm. His serum sodium is 125 mmol/L (135-145) and his serum potassium is 6.0mmol/L (3.5-4.5). His serum creatinine is 150micromol/L (50-100).

Questions 1. What is the probable diagnosis? 2. What investigations would you do prior to initiating treatment? 3. What is your initial treatment? 4. If the diagnosis is confirmed what should the ongoing management be? 5. What advice do you give him about how to cope with intercurrent illnesses such as respiratory and

gastrointestinal illnesses? 6. What are the possible causes of this problem? 7. What other investigations should you consider?

Case 3 (relates to Hyperthyroidism [Endo9]) A 56 year old Japanese woman presents to the medical team at the hospital after being referred by her GP. She has lost 7 kg in weight despite a good appetite, feels irritable and always wears shorts because she gets too hot. She has noted a shake in her hand that affects her writing. She is mildly breathless on exertion. On examination she has a heart rate of 120/min which is irregularly irregular. She has warm peripheries and a fine tremor. There is diffuse enlargement of the thyroid gland and a bruit is heard over the gland. On examination of her eyes there is a full range of ocular movements but she has lid lag and lid retraction.

Questions 1. What is the probable diagnosis? 2. What investigations would you order?

If your diagnosis proves to be correct, what might your initial treatment be for: 3. her cardiac rhythm? 4. her underlying problem? 5. Are there adverse effects you would need to watch for? What advice would you give to the patient? 6. How would you monitor her response to treatment? 7. How long would you continue treatment for?

She responds to treatment but when you discontinue it, she relapses several months later. 8. What would you do next?

Tutorial 8 Clinical Pathology

Lead academic advisors: Prof Peter Browett email: [email protected]

26

Dr Laura Young email: [email protected]

Resources and revision: Clinical Scenarios as below http://mbchb.auckland.ac.nz/scenarios Phase 1 materials e.g. from Principles of Medicine; Blood, Immunity and Infection, Regulation of Body Function courses Year 4 Clinical Pathology Teaching from Formal Learning Weeks

This tutorial aims to highlight the diagnosis and management of clinical problems from a laboratory perspective. The focus is on identification of abnormal laboratory results, forming a differential diagnosis and further testing to identify the cause. Two nominated students in each cohort will organise for each of the five cases to be assigned so that two students research each case and then lead the discussion.

Case 1 (relates to Hyponatraemia [CB1]) A 75 year old woman is taken to her GP with confusion. Relevant clinical findings include normal jugular venous pressure (+2cm), moist mucous membranes and no peripheral oedema with clear lung fields on auscultation. She has no recent history of medical illness but was commenced on citalopram one month ago because of symptoms of depression.

PLASMA units ref. range Na 127 mmol/L 135-145 K 4.2 mmol/L 3.5-5.0 Cl 92 mmol/L 95-110 Creatinine 52 µmol/L 45-90 Urea 4.0 mmol/L 3.2-7.7 Glucose 7.0 mmol/L 3.0-5.6

(fasting) Urate 0.18 mmol/L 0.2-0.36 Osmolality (measured) 271 mOsmol/kg 280-300 Calculated osmolality 265 mOsmol/kg 280-300 URINE Na 65 mmol/L diet-

dependent Osmolality 620 mOsmol/kg 50 - 1200

Questions 1. List the causes of hyponatraemia under the following headings:

- primary sodium loss - primary water overload - other

2. Is this hyponatraemia likely to be acute (<48h) or chronic? Is there a difference in the management of acute vs. chronic hyponatraemia? – explain.

3. What can we deduce from the urine osmolality? What causes of hyponatraemia can be excluded with this information?

4. Give a formula for the calculation of plasma osmolality. Under what conditions can there be a large discrepancy (>10 mOsmol/kg) between measured and calculated plasma osmolality?

5. Of the remaining causes of hyponatraemia, which can be excluded, and which is the most likely? Do any further tests need to be done?

6. Why is plasma Na often low in patients with hyperglycemia? 7. What commonly causes acute hyponatraemia in surgical patients and how is it managed? 8. Hypernatraemia is also a cause of confusion. What are the common causes of this biochemical abnormality and how is it managed?

Case 2 (relates to Hypercalcaemia and back pain [CB2]) A 58 year old woman has presented to her GP with frequent attacks of purulent sinusitis over a four year period. During this time she has also suffered from three episodes of pneumonia. She is a non-smoker and

27

does not suffer from allergies. CT scan of the sinuses revealed pansinusitis. High resolution CT scan of the chest revealed bronchiectatic changes in the right middle lobe. Serum immunoglobulins were measured.

IgG 2.6g/L (7.0 - 16.0) IgA <0.3g/L (0.4 - 2.5) IgM <0.3g/L (0.8 - 4.0) Serum protein electrophoresis:

Marked reduction in gamma region.

No monoclonal bands.

Questions 1. What are the abnormalities and their significance? 2. What other investigations are necessary? 3. What is the likely diagnosis? 4. How should this woman be managed?

Case 3 (relates to Haemochromatosis [MG1]) A 48 year old woman is seen by her GP complaining of fatigue. A series of investigations are arranged including the following:

Serum iron 30 µmol/L (10-30) TIBC 40 µmol/L (35-75) % Saturation 75% (20-55%) Ferritin 450 µg/L (15-300) ALT 43 U/L (<45)

Questions 1. What is the significance of these results? 2. What is the most likely molecular defect in this disorder, how does the mutation cause the disease

and which factors affect the penetrance (expression) of the abnormal gene. 3. What is the role of genetic testing in this individual? 4. What is the pattern of inheritance of this condition, what additional family members should be

screened, and what screening tests would you use? 5. What organs may be involved in advanced cases of this disease? 6. How would you manage this patient?

Case 4 (relates to Child with sore joints [Paed12]) A 12 year old girl presents to the ED with a painful swollen right knee. She has a temperature of 38.1°C. She reports a sore throat about three weeks ago, which was not investigated or treated.

Questions 1. What other aspects of the history and examination would be important and what tests would you arrange? The following results are obtained:

Anti streptolysin 325 IU/ml (0-75) Anti DNase B 680 IU/ml (0-75) C-reactive protein 54 mg/L (0 - 8) Peripheral blood culture No growth after 4

days of incubation

2. What is the most likely diagnosis and what other differential diagnoses did you consider? 3. Describe the criteria for making this diagnosis. How is it managed? Three years later she presents to hospital with worsening shortness of breath, fever and lethargy for three weeks’ duration. Her temperature is 38⁰C and an early diastolic murmur is heard on expiration, at the left sternal edge. Three sets of peripheral blood cultures are taken as part of evaluation during the first two hours of hospital admission and these are all growing a gram positive coccus.

28

4. What is the probable diagnosis? 5. Describe the criteria for making this diagnosis. How is it managed? 6. Describe the circumstances when antibiotics should be given to prevent this illness.

Case 5 (relates to Hypercalcaemia and back pain [CB2]) A 55 year old woman presents to the ED with severe back pain exacerbated by movement in the region of her mid thoracic spine. She has smoked 10 cigarettes per day for 20 years and has a noticed recently that she passes urine more often, including getting up twice at night. She has a past history of excessive alcohol intake and hypertension.

On initial physical examination, her BMI is 22. Heart rate 100/min, respiratory rate 15/min. She is pale with tenderness over her spine between her scapulae. There is no finger clubbing. A lateral chest X-ray shows a crush fracture of the fourth thoracic vertebra.

Plasma units ref. range Calcium (total) 3.30 mmol/L (2.10-2.55) Phosphate 1.39 mmol/L (0.7-1.5) Total protein 93 g/L (62-80) Albumin 33 g/L (35 - 47) ALP 92 U/L (<120) Urea 14.7 mmol/L (3.0-8.0) Creatinine 170 µmol/L (45-90) Haemoglobin 101 g/L (115-160) MCV 82 fL (76-96) MCH 29 pg (27-32)

Questions 1. What are the common causes of hypercalcaemia? For each cause, describe the pathophysiological mechanism for the hypercalcemia . 2. How do you use albumin and phosphate levels in interpreting a serum calcium result? Can ionised calcium be measured directly? 3. What diagnoses are likely in this patient? What other tests would you request to differentiate them and how would you interpret these tests? 4. Why might this patient’s urea and creatinine concentrations be increased? 5. What treatments may be useful for this patient’s hypercalcaemia?

29

Tutorial 9 Rheumatology

Lead academic advisors: Prof Fiona McQueen email: [email protected] Assoc Prof Nicola Dalbeth email: [email protected]

Resources and revision: Clinical Scenarios as below http://mbchb.auckland.ac.nz/scenarios Phase 1 materials e.g. from Principles of Medicine; Blood, Immunity and Infection, Musculoskeletal courses Year 4 Rheumatology Teaching from Formal Learning Weeks Resources on the portal – Formal learning and Year 4

Two nominated students in each cohort will organise for each of the four cases to be assigned, so that two students research each case and then lead the discussion.

Paper Patients

Case 1 (relates to Swollen and tender joints [Rh2]) Paula REUBENS - Presenting History A 28 year old secretary presents with a complaint of morning stiffness and pain in the fingers for about one week. On further questioning, she mentions having some soreness and swelling of her wrists about 3 months ago. This followed an attack of ‘flu’. She did not see her doctor but thought she had ‘arthritis’, took some Nurofen during her summer holiday, and everything cleared up in about 2 weeks. The patient is a solo parent with a 7 year old child. She is worried about her working future.

Questions 1. What do you understand by the word ‘arthritis’? 2. What types of disorder might account for this patient’ symptoms? 3. Does the presence of ‘morning stiffness’ help you in your evaluation of this problem? 4. Can you give her any advice that would help her deal with anxiety about the future?

Further History She returns 1 month later with obvious swelling of both knees. She noticed they were particularly painful on activities such as ascending and descending flights of stairs. The soreness of her hands has increased and she has obvious swelling of several metacarpophalangeal and proximal interphalangeal joints. In addition, she has also noticed tingling in her right hand waking her at night and also during the day after knitting or crocheting. In the past she has had rheumatic fever and an appendicectomy, but she has never been told she has a heart murmur. There is no significant family history of gout or arthritis but her paternal grandfather walked with a stooped posture and had, as a youth, backache which he blamed on long hours in damp trenches in the 1st World War. He walked with a stiff gait and constantly complained of his ‘arthritics’.

5. Is there anything new that helps you solve the patient’s problems? 6. What do you think of the complaint of tingling in the fingers and can anything be done to relieve it? 7. Is the history of rheumatic fever significant? 8. Is there any relevance to her grandfather’s history?

Further History Over the next month her shoulders, wrists and feet become involved by stiffness and swelling. She also complains of pain in the back of the head and in front of her ear for 1 month. She is very tired. She is very worried about keeping her job and she has also noticed she is becoming increasingly bad tempered with her son and at work.

9. Why is she having pain in the back of her head and ear? 10. What effect does chronic pain have on mood (and vice versa)? 11. Why is she tired?

Physical Examination On examination she appeared a fit, but thin lady. Her pulse was 80/minute and regular, BP was 150/80. The musculoskeletal examination revealed tenderness and swelling at 3rd and 4th PIP joints of the left hand and 2nd PIP of right hand. At the MCP joints there was also slight swelling of 2nd and 3rd bilaterally. There was no swelling at the wrists but pain on full extension on the right side (which was limited by 200 compared to left). Examination of the feet revealed tenderness on lateral squeeze test at the MTPs

30

bilaterally. Examination of the spine (axial skeleton) was normal. She had a small nodule at her left elbow, and reduced light touch sensation of the skin of the palm over the thenar eminence, and index finger.

12. What do you consider are the most likely forms of arthritis in this lady? 13. What are the clinical features which are consistent with your first diagnosis? List at least five. 14. What laboratory tests and X-Rays would be indicated in this patient 15. How would you make an overall assessment of disease activity in this patient? 16. What is your initial treatment for her acute arthritis? 17. How would you decide whether to initiate treatment with a DMARD? 18. If you did start treatment with a DMARD, which medicine would you choose? 19. What precautions would you take in monitoring for any adverse effects? 20. If your initial treatment is ineffective, what other options are there? Are there any adverse effects

associated with these treatments?

Case 2 (relates to Inflammatory low back pain [Rh7]) Michael Slater - Presenting History A 25 year old man presents to his GP with low back pain. The pain has been present for more than 18 months but increased in intensity over the last 3 months. There is pronounced early morning stiffness lasting over 2 hours and nocturnal pain such that he is awoke most nights from sleep. He has tried painkillers & ibuprofen has helped his symptoms.

Questions 1. What are the characteristics of inflammatory back pain? 2. What else would you ask him about?

Examination shows tenderness and swelling of the insertion of the Achilles tendon, swelling and warmth of the right knee with a moderate joint effusion, restricted internal rotation of both hips, and tenderness of both sacroiliac joints. Schober test measures 15 to 18cm.

3. Demonstrate: • Examination for a knee effusion • Assessment for SI joint tenderness • Measurement of the Schober test

4. What investigations would you request? 5. What is the diagnosis? (Justify your answer) 6. What is your initial treatment?

He returns to see you again four months after starting your initial treatment. He has ongoing back pain and is having difficulty with many of his daily activities, including showering, getting up stairs and bending over. He is stiff in the morning for more than three hours. He is concerned that he may lose his job because he has required three weeks of sick leave in the last two months.

7. What other treatment options should you consider?

Case 3 (relates to SLE / fatigue / pain and swelling in the hands [Rh3]) Miss Tina Falofa - Presenting History An 18 year old Samoan woman presents with arthralgia involving her hands, wrists and ankles. On interrogation she also admits to Raynaud’s phenomenon, mouth ulcers and a photosensitive skin.

Questions 1. What is the most likely diagnosis? Give a differential diagnosis. 2. What is the most common cause of Raynaud’s phenomenon and what are the clinical features? 3. What other features on history would you question her about?

On examination She appears well but has a mild malar rash. There is spindling of the PIP joints with tenderness and some reduction in grip strength There is reduced air entry to the left lower lobe

• possibility of pleural effusion / pneumonia – will need investigation • other features consistent with lupus – often not very dramatic clinical findings

Investigations Her Hb is 105 g/L and her platelet count is 80 x 109/L.

4. What else do you want? Management

5. How would you manage her fatigue and arthralgia?

31

Progress Subsequently, she develops a persistent skin rash on her face and arms and a polyarthritis involving her MCP joints and wrists.

6. What other treatment options should you consider? 7. What adverse effects do you need to be aware of with this treatment?

One year later she has an abrupt rise in creatinine from 70 micromol/L to 160 micromol/L A mid-stream urine shows proteinuria and microscopic haematuria.

8. What is the reason for this? 9. What would you treat it? 10. What can be done to reduce the adverse effects of the treatment?

Case 4 (relates to Knee pain [M20] and Gout / acute pain and swelling of knee [Rh5]) Ethel BENT - Presenting History: A 63 year old lady, a pillar of the local community, comes to see you complaining of pain in both knees. Last Saturday she ran a cake stall for 8 hours outside the shopping centre and the following morning she could hardly move. She had noted some anterior knee discomfort on climbing up and down stairs or while kneeling in the garden over the last few years, particularly during the winter, but never as bad as this. She has always had a weight problem, but her current weight of 89kg has remained unchanged for some years. Her New-Age daughter-in-law has recently started her on a vegetarian diet with celery and boron supplements.

She also remembers she was a rep netball player in her youth and her career was foreshortened by ‘knee troubles’.

Questions 1. What would be the commonest arthritis in this community affecting both knees of a 63 year old

female? 2. What factors in the history may have contributed to this and to her current presentation? 3. She asks for your opinion concerning her diet?

On Examination She had small effusions in both knees. She was tender over the medial joint line on each side. Her collateral and cruciate ligaments were intact and she had a full range of movement. There was some patellofemoral crepitus. Her feet and ankles were normal. In her hands there was some bony enlargement of the IP and PIP joints of the fingers of both hands and at the bases of both thumbs, particularly in her dominant right hand. These joints were not painful.

4. Would her past history of ‘knee trouble’ be significant? 5. What was the likely nature of her knee effusions? 6. You realize afterwards you forgot to examine her hips. Does this matter? 7. What is the nature of the swelling in her finger joints? If this is not painful could it be arthritis? 8. What other organ systems should you assess in trying to make your rheumatological diagnosis? 9. Would a nonsteroidal anti-inflammatory drug (NSAID) have been more effective than her

paracetamol? 10. Why was her x-ray taken ‘weight bearing’? 11. What blood tests, if any, would it be reasonable to request in this patient? 12. Would you regard Tiger Balm as an acceptable treatment? 13. Are there any dangers in her using a knee support? 14. Could a stick have been helpful for her?

Unfortunately her right knee continues to give problems and over the next two years becomes more and more painful, not only when weight bearing but also at rest. She has to give up her community work and has difficulty looking after their two-storied house.

15. What additional forms of treatment may be appropriate to consider? When?

QQQuuuiiizzz 1. A woman of 65, with rheumatoid arthritis involving many joints, suddenly develops more severe

pain swelling and heat in the right knee, but not in the other joints. Would you: a) Increase her NSAIDs for pain relief and put her to bed? b) Inject the knee with Cortisone?

32

c) Take her temperature and aspirate the knee to relieve pressure and look for monosodium urate crystals?

d) Request urgent full blood count, ESR and x-ray? 2. A woman of 50 with rheumatoid arthritis of 10 years duration on prednisone for 5 years present

with ankle pain having twisted it slightly stepping off the pavement. The ankle is swollen as it has been for some time but maximum tenderness is 5 cm above lateral malleolus.

a) What would you suspect?

b) What would you do?

3. A woman of 25 presents with a painful warm swollen knee of spontaneous onset and 3/52 duration.

Having established there was no history of injury, other arthritis, or evidence of systemic illness, what tests would you do?

4. A patient presenting with polyarthritis and involvement of DIP joints of fingers may have (more than

one): a) Osteoarthritis (OA) b) Osteoarthritis with gout c) Psoriatic arthritis (PsA) d) Rheumatoid arthritis (RA) e) RA + OA

5. A patient presenting with polyarthritis with wrist involvement is unlikely to have OA? (True/False) 6. A man of 25 who does heavy lifting at work presents with low back ache. What questions help you

distinguish an inflammatory from a mechanical cause?

7. A Māori man age 45, taking part in veterans rugby, playing in borrowed boots presents after the

game with a painful great toe. He may have gout, but what else may he have?

8. What are the cardinal signs of inflammation, and what is the symptom of inflammation which one

should always ask about?

33

9. To detect early erosive change in a patient with arthritis is it better to x-ray small joints or large

joints?

10. A fit man of 40 from a Pacific Island, who is moderately overweight, presents with aching in the

shoulder preventing him sleeping. His serum urate is 0.54 mmols/L. What is the differential diagnosis?

11. Nodules on the elbow are found in association with:

a) Gout b) Psoriatic arthritis c) SLE d) Rheumatoid arthritis e) Rheumatic fever

12. Which of the following are more common in Pacific peoples compared with Europeans?

a) SLE b) Psoriatic arthritis c) Rheumatoid arthritis d) Rheumatic fever e) Gout f) Ankylosing spondylitis

34

Tutorial 10 Infectious Diseases (Medicine)

Lead academic advisor: Assoc Prof Mark Thomas email: [email protected]

Resources and revision: Clinical Scenarios as below http://mbchb.auckland.ac.nz/scenarios Phase 1 materials e.g. from Principles of Medicine; Blood, Immunity and Infection, Year 4 ID Teaching from Formal Learning Weeks

Suggested format: This session for Year 4 students in medical half year focuses on clinical and laboratory diagnosis, and treatment of meningitis and pyelonephritis. Penicillins, cephalosporins, trimethoprim and ciprofloxacin will be discussed during the session.

Two students will be allocated to lead the session, and bring along any questions. All students will be expected to have read and considered the two case scenarios before attending the session. They will also be provided with access to resources that they might use to inform themselves before attending the session.

Meningitis

Case 1 (relates to Unexplained loss of consciousness [ED3], The agitated and combative patient [ED4], Sudden onset severe headache [ED7], Falls and collapses [ED12], Hearing loss [ORL1], Altered level of consciousness in a child [Paed7], Unwell neonate [Paed25], Congenital hearing loss [Paed29], Acute psychosis [Psych4], Delirium / dementia [Psych8], and Medically unexplained symptoms [Psych10]) A 68 year old man with no fixed abode presents to the Emergency Department agitated and aggressive and threatening to punch you and the nursing staff. He has a fever of 38.7C, is confused, has a degree of neck stiffness, but no rash. A CSF sample shows:

WBC = 3,500 x 109/l (80% polymorphs) RBC = 50 x 109/l Protein = 3.5g/l (n = 0.15 – 0.45) Glucose = 0.8 mmol/l (n = 2.8 – 4.4) No organisms are seen in the CSF.

Questions: 1. What are the most likely causes of his illness? 2. What clinical features will be useful pointers to the aetiology? 3. How do you interpret the CSF results? 4. What further information will be available, and when, from the microbiology lab? 5. Should he be treated now, or should you await further results? 6. If you decide to treat him now, what drug should be used? 7. What drug would you use if the microbiology lab reported seeing Gram positive cocci in his CSF? 8. Why would you use that drug?

Other topics which enthusiastic students might consider: 1. Aetiology of meningitis in special clinical scenarios e.g.

• in neonates • following neurosurgery or head injury • in immunocompromised patients

2. How to perform a lumbar puncture Watch the NEJM Procedures in Medicine video at: http://www.youtube.com/watch?v=-gX2nJMi20Q 3. The role of intravenous steroid therapy given before starting antimicrobial therapy to reduce adverse

outcomes Dexamethasone 10mg 6hrly IV for 4 days, started 15-20minutes before starting antimicrobial therapy improves the outcomes in patients with meningitis due to Streptococcus pneumoniae. See: N Engl J Med 2002;347:1613-5.

35

Pyelonephritis

Case 2 (relates to Acute abdominal pain [Abdo2], and Flank pain and dysuria [GU6]) A 48 year old lady attends Emergency Department with a two day history of fevers and chills. Her observations are concerning: blood pressure of 90/60mmHg, pulse 120 beats per minute, temperature 39.7°C and oxygen saturations 94% on air. You are working as the House Officer in the department and are asked to go and see her by the nurse looking after her. She reports five days urinary frequency and dysuria. She saw another doctor three days ago who tested her urine sample with a dipstick but did not send the sample for microscopy or culture. He gave her treatment with trimethoprim. You find that she has left loin pain tenderness. You request an MSU which shows 160 WBC x 106/L, 50 RBC x 106/L, 10 epithelial cells x 106/L, and profuse bacteria.

Questions 1. What are the clinical features of cystitis and pyelonephritis? 2. Should all patients with symptoms suggestive of cystitis have an MSU examined? 3. What are the six most likely pathogens? 4. What drugs are commonly prescribed for cystitis and for pyelonephritis? 5. Are any of these drugs active against all of the usual pathogens? 6. Why might you choose not to use the drug with the broadest spectrum of activity as initial treatment

for all your patients with cystitis? 7. What antimicrobial treatment is appropriate for this woman now? 8. Any other investigations?

Other topics which enthusiastic students might consider: 1. How to collect an MSU sample for microscopy and culture See:

http://www.youtube.com/watch?v=S49nUD-iA4A 2. The much lower rates of resistance in bacteria from a representative sample of patients with UTIs

compared with the bacteria from patients who are commonly selected for testing of an MSU specimen.

See: Antibiotic resistance in uncomplicated urinary tract infection: problems with interpreting cumulative resistance rates from local community laboratories. NZMJ 2002 (Jan 25);115(1146):12-4.

36

Tutorial 11 Haematological Cancer Presentations in the Community

Lead academic advisors: Prof Peter Browett email: [email protected] Dr Laura Young email: [email protected]

Resources and revision: Clinical Scenarios as below http://mbchb.auckland.ac.nz/scenarios Phase 1 materials e.g. from Principles of Medicine; Blood, Immunity and Infection courses Cancer Continuum Module Year 4 Haematology Teaching from Formal Learning Weeks

The focus is on typical presentations of chronic haematological malignancies in the community, and the differential diagnosis and laboratory testing of these. Two nominated students in each cohort will organise for two students to research each case and be prepared to lead the discussion.

Case 1 (relates to Polycythaemia [Blood6]) A 64 year old male presents with a history of headaches of recent onset. On examination he is plethoric but there are no other abnormal findings. Investigations include the following full blood count:

Full blood count Red cell count 7.9 x1012/L 4.5-6.5 Haemoglobin 190 g/L 130-175 PCV 0.63 0.4-0.52 MCV 79 fL 80-99 MCH 26 pg 27-33 Red cell morphology abnormal ESR 1 mm in 1 hr 1 - 10 Platelet count 420 x109/L 150-400 White cell count 8.9 x109/L 4.0-11.0

Segmented neutrophil 6.27 x109/L 2.0-7.5

Basophil 0.22 x109/L 0-0.2 Eosinophil 0.3 x109/L 0-0.5 Monocyte 0.26 x109/L 0-0.8 Lymphocyte 1.85 x109/L 1.5-3.5 Blood film

Red cells - Increased numbers of microcytes and hypochromic cells

Questions 1. Describe the important abnormalities in this result 2. (a) List the possible causes of the raised haemoglobin in this patient

(b) What other features of the history and examination would be important in differentiating these? 3. What tests would you arrange? 4. What is appropriate initial management?

Case 2 (relates to Polycythaemia [Blood6]) A 30 year old woman is evaluated for the purposes of insurance. She has been well and has no relevant past history of note. Routine investigations are arranged and the following full blood count result is received:

37

Full blood count Red cell count 3.9 x1012/L 3.6-5.6 Haemoglobin 121 g/L 115-155 PCV 0.37 0.35-0.46 MCV 82 fL 80-99 MCH 29 pg 27-33

Red cell morphology normal

Platelet count 950 x109/L 150-400 White cell count 9.2 x109/L 4.0-11.0

Segmented neutrophil 6.27 x109/L 2.0-7.5

Basophil 0.14 x109/L 0-0.2 Eosinophil 0.3 x109/L 0-0.5 Monocyte 0.26 x109/L 0-0.8 Lymphocyte 2.23 x109/L 1.5-3.5 Blood film

Platelets: Platelet numbers are significantly increased with some large and occasional giant forms seen

Questions 1. What is the main abnormality and what is the differential diagnosis? 2. What additional laboratory tests would help you confirm or exclude the differentials in your list? Reactive causes are excluded and the patient is assessed by haematology. 3. How would she be investigated and managed? After three years of routine haematology follow up, this patient wishes to consider pregnancy. Her platelet count is 1050 x 109/L. 4. What advice would you give and (briefly) what therapy could be considered?

Case 3 (relates to Polycythaemia [Blood6]) A 55 year old man presents with a 3 month history of increasing fatigue and early satiety after meals. He has lost 3 kilograms. On examination the spleen is palpable 12 cm below the costal margin. There is no lymphadenopathy, and the liver is of normal span. A full blood count reveals the following:

38

Full blood count Red cell count 3.5 x1012/L 4.5-6.5 Haemoglobin 105 g/L 130-175 PCV 0.3 0.4-0.52 MCV 87 fL 80-99 MCH 30 pg 27-33 Red cell morphology abnormal Nucleated rbc 0.35 x109/L Reticulocyte count 95 x109/L 10-100 Platelet count 140 x109/L 150-400 White cell count 17.1 x109/L 4.0-11.0 Myelocyte 0.81 x109/L Metamyelocyte 2.23 x109/L Band neutrophil 2.9 x109/L 0-0.6

Segmented neutrophil 7.4 x109/L 2.0-7.5

Basophil 0.31 x109/L 0-0.2

Eosinophil 0.4 x109/L 0-0.5 Monocyte 1.2 x109/L 0-0.8 Lymphocyte 1.85 x109/L 1.5-3.5 Blood film

Red cells: Nucleated red cells are noted. Polychromatic cells are increased. Many poikilocytes and tear drop cells are present.

White cells: Immature myeloid forms are noted

Platelets: Increased numbers of giant platelets

Questions 1. What is the differential diagnosis of splenomegaly in a patient like this? 2. What are the abnormalities on the blood film?

Has this combination of abnormalities got any special significance? 3. What is the likely diagnosis in this case? 4. What additional investigations would be helpful?

39

Case 4 (relates to Lymphadenopathy and splenomegaly [Blood5]) A 65 year old man with osteoarthritis of the hip is evaluated for hip arthroplasty surgery and routine preoperative investigations are undertaken. He is otherwise well with no significant background history.

Full blood count Red cell count 3.5 x1012/L 4.5-6.5 Haemoglobin 119 g/L 130-175 PCV 0.34 0.4-0.52 MCV 87 fL 80-99 MCH 30 pg 27-33 Red cell morphology normal Platelet count 140 x109/L 150-400 White cell count 24.2 x109/L 4.0-11.0

Segmented neutrophil 6.1 x109/L 2.0-7.5

Basophil 0.1 x109/L 0-0.2 Eosinophil 0.4 x109/L 0-0.5 Monocyte 0.6 x109/L 0-0.8 Lymphocyte 12.7 x109/L 1.5-3.5 # 4.3 x109/L Blood film White cells: # = smear cells. Increased numbers of mature lymphocytes are seen.

Questions 1. Describe the abnormalities seen. What are the causes of a lymphocytosis? What is the most likely

cause? 2. What signs would you look for on examination? 3. What additional investigations would you arrange? 4. Does this patient currently require treatment? Can he have his operation? What is his long-term

outlook? Two years later he is evaluated because of increasing fatigue and shortness of breath. Additional investigations are arranged. His haemoglobin is now 75g/L; white cell count 29.6x109/L. On the blood film spherocytes and polychromatic cells are present and his serum haptoglobin is markedly reduced. A direct antiglobulin test (DAT; direct Coomb’s test) is positive (anti-IgG). 5. What haematological abnormality has this patient now developed? 6. Comment on the association of this abnormality with the haematological disorder that we had

already identified. 7. What is the other immune-mediated disorder that affects another cell lineage that can also be

associated with the underlying haematological disorder?

40

Tutorial 12 Dermatology

Lead academic advisor: Dr Paul Jarrett email [email protected]

Resources and revision: Phase 1 materials e.g. from Principles of Medicine; Musculoskeletal Year 4 Dermatology Teaching from Formal Learning Weeks Asynchronous Learning Module Dermatology on the MBChB portal

First hour – Dermatology Treatments This tutorial follows on from the teaching in the Formal Learning weeks. It is done in the same style and based around the Clinical Scenarios. The material will be presented by tutors, and made available on the designated dermatology website.

Second hour – Clinical Reasoning The two nominated students at the site will have the tasks of:

• Each finding a case with a dermatological problem, from their ward experiences. Present key features to group (suggest 0-4 Powerpoint slides max.).

• Compiling questions from students relating to the cases or dermatology.

Students will be expected to communicate with each other in order to prepare for the tutorial. After the presentation of the two patient vignettes (max 15 minutes), the remainder of the hour will be spent on:

1. discussing relevant aspects of the two cases and covering student queries 5. clinical reasoning:

a. how typical or atypical were the presentations; b. specific history and exam findings, and techniques; c. common differential diagnoses; d. red flags, pitfalls; e. the role and interpretation of common investigations; f. choosing treatments / delivery mechanisms.

6. addressing the broader curriculum domains of Hauora Māori and Population Health 7. covering any other specific topics.

41

Tutorial 13 Infectious Diseases (Surgery)

Lead academic advisor: Assoc Prof Mark Thomas email: [email protected]

Resources and revision: Clinical Scenarios as below http://mbchb.auckland.ac.nz/scenarios Phase 1 materials e.g. from Principles of Medicine; Blood, Immunity and Infection, Year 4 ID Teaching from Formal Learning Weeks

Suggested format: This session for Year 4 students in surgical half year focuses on clinical and laboratory diagnosis, and treatment of faecal peritonitis and septic arthritis. Gentamicin, metronidazole are discussed along with other antibiotics.

Two students will be allocated to lead the session, and bring along any questions. All students will be expected to have read and considered the two case scenarios before attending the session. They will also be provided with access to resources that they might use to inform themselves before attending the session. Faecal peritonitis Case 1 (relates to Postoperative complications [Abdo22] and Acute abdominal pain in children and adolescents [Paed5])

75 year old woman had an anterior resection (large bowel operation) 4 days ago. She develops fever, abdominal pain and has not opened her bowels since surgery, she suffers from diabetes. Her heart rate is 130 beats per minute, temperature is 38.3°C and she is breathing rapidly. The nursing staff are worried about her. You are the on call house officer and are asked to review her.

You find that she has no signs of pneumonia, intravenous cannula infection or deep vein thrombosis. She does have a rigid abdomen with guarding and rebound tenderness, plus absent bowel sounds.

Questions

1. What is the differential diagnosis in this woman? 2. What organism(s) are likely to be responsible for her septicaemia? 3. What regimen might you choose as treatment for her? 4. The consultant suggests a regimen containing gentamicin. How should this drug be administered,

and in what dose? 5. What are the most likely adverse effects of gentamicin and how might you detect them? 6. You decide to treat the patient with metronidazole as well. Explain why, and how you would

administer the drug. Septic arthritis Case 2 (relates to Osteomyelitis [Paed1], Child with abnormal gait [Paed3], Child with sore joints [Paed12], Febrile infant [Paed24], Reactive arthritis [Rh1], Swollen and tender joints [Rh2], Gout / acute pain and swelling of knee [Rh5], Knee injury [m12], and Knee pain [M20])

A 63 year old woman had a left total hip joint replacement inserted at surgery 9 weeks ago, because of increasing pain with walking over the last 3 years. She now presents with redness of the skin over the joint, pain and swelling in the knee, and discharge of a small amount of pus from the wound, for the last three days. Microscopy of a sample of pus discharging from the wound shows profuse neutrophils and Gram positive cocci in clumps. Culture of this sample has isolated a pure, heavy growth of a methicillin resistant Staphylococcus epidermidis.

Questions 1. What is the likely diagnosis? 2. What organisms are the most common cause of this type of infection? 3. How will you make a diagnosis? 4. What treatment do you advise?

42

Tutorial 14 ORL

Lead academic advisor: Mr Richard Douglas email [email protected]

Resources and revision: Clinical Scenarios as below http://mbchb.auckland.ac.nz/scenarios Phase 1 materials esp. Head and Neck anatomy from Medical Neuroscience Year 4 Teaching from Formal Learning Weeks Asynchronous Learning Module (will be uploaded Jan 2013)

Format: Two hour session for 4th year students in surgical half of the year. The two hour session will cover the five most common ORL presentations students will encounter in their emergency, general practice and ORL placements. Students are expected to have read and considered the cases prior to coming to the session.

Prior Reading: For review please see the University of Auckland ORL website (available from February 2013). The following references are also very useful: Case 1: Schlosser RJ. Epistaxis. N Engl J Med. 2009;360: 784-9. Case 2: Osguthorpe JD, Nielsen DR. Otitis Externa: Review and Clinical Update. Am Fam Physician.2006:1;74(9):1510-16. (http://www.aafp.org/afp/2006/1101/p1510.html) Case 3: http://emedicine.medscape.com/article/837048 Case 4: http://emedicine.medscape.com/article/1159385 Case 5: Fook L, Morgan R. Hearing impairment in older people: a review. Postgrad Med J. 2000;76:537-41. (http://pmj.bmj.com/content/76/899/537.full)

Case 1 Epistaxis (relates to Nasal disease [ORL2]) A 60 year old man presents to ED with ongoing left-sided epistaxis. He has had no previous episodes of epistaxis or easy bruising.

Questions 1. What is the relevant anatomy? 2. What are the important points to explore in the history? 3. How would you examine the patient? 4. What is the treatment?

Case 2 Otalgia/otorrhoea (relates to Hearing loss [ORL1], and Febrile infant [Paed24]) A 20 year old returns from holiday in Samoa where he swam in still water. He complains of a two day history of worsening otalgia and otorrhoea.

Questions 1. What is the relevant anatomy? 2. How would you examine this patient? 3. What is the differential diagnosis? 4. What is the management of otitis media and otitis externa? 5. What are the complications of otitis media and externa?

Case 3 Sore throat, tonsillitis, paratonsillar abscess and epiglottitis (relates to Upper respiratory tract symptoms [Resp07], Upper airway disease [ORL4], and Compromised airway [ED1]) A 10 year old male presents to ED with a severe sore throat that has been worsening for 2 days. He is now unable to swallow and the parents state he has begun to drooling profusely.

Questions 1. What is the relevant anatomy? 2. What other important questions would you ask on history? 3. How would you examine this patient? 4. What is the differential diagnosis? 5. What is the management of the conditions discussed in the differential diagnosis?

43

Case 4 Tinnitus and vertigo (relates to Dizziness and vertigo [N8], and Hearing loss [ORL1]) A 70 year old female presents to the Emergency Department with sudden onset vertigo and tinnitus.

Questions 1. Briefly, what is the relevant anatomy and physiology? 2. How would you examine this patient? 3. What is the differential diagnosis? 4. What is the management of the common diseases discussed?

Case 5 Hearing loss (relates to Hearing loss [ORL 1], and Congenital hearing loss [Paed29]) A 50 year old man is brought to his family doctor by his wife who complains that he cannot hear the TV unless he turns the TV up loudly. He is an auto-mechanic specializing in large truck engines.

Questions 1. What are the important points to gather in the history? 2. How would you examine this patient? 3. What are the different types of hearing loss? 4. What is the differential diagnosis? 5. What are the investigations you would ask for?

Case 6 Hoarseness of voice (relates to Upper respiratory tract symptoms [Resp07], and Upper airway disease [ORL 4]) A 55 year old man presents with a 1 month history of worsening hoarseness of voice. He has a 35 pack year smoking history.

Questions 1. What are the important points to gather in the history? 2. How would you examine this patient? 3. What are the common and important causes of hoarseness of voice? 4. What are the investigations you would ask for?

44

Tutorial 15 Plastics

Lead academic advisor: Mr Jonathan Burge email [email protected]

Resources and revision: Clinical Scenarios as below http://mbchb.auckland.ac.nz/scenarios Phase 1 materials e.g. from Principles of Medicine, Musculoskeletal

Application of Basic Principles of Plastic Surgery to Clinical Cases

• 30 minute lecture. Topics covered will be: o basic principles of reconstruction o the scope of plastic surgery including its involvement in burns, upper limb surgery,

breast surgery, aesthetics, head & neck, craniofacial, cutaneous malignancy and trauma

• 45-60 minutes to assess patients and discuss in small groups then present what they have concluded.

• 30 minutes discussion between lecturer and the medical students. All questions the students have related to plastic and reconstructive surgery will be discussed as an open forum